Nilpotent Matrix

$\spadesuit$Generalized Eigenspace $\spadesuit$

Let $A$ be $n$-square matrix and $W$ be a subspace of ${\mathcal C}^{n}$. For any vector ${\mathbf x}$ in $W$, if $A{\mathbf x} \in W$, then the subspace $W$ is called an A - invariant.

We write the characteristic polynomial in terms of multiplicity of characteristeic values.

$\displaystyle \Phi_{A}(t) = (t - \lambda_{1})^{n_{1}}(t - \lambda_{2})^{n_{2}} \cdots (t - \lambda_{r})^{n_{r}}$ (5.1)

$\displaystyle \lambda_{i} \neq \lambda_{j}  (i \neq j),  n = n_{1}+n_{2}+\cdots+n_{r} $

Here, for the eigenvalues $\lambda_{i}(1 \leq i \leq r)$, we let

$\displaystyle W(\lambda_{i}) = \{{\mathbf x} \in {\mathcal C}^{n} : (A - \lambda_{i}I )^{j} {\mathbf x} = {\bf0}\} $

Then the following holds.

$\displaystyle V(\lambda_{i}) = W_{1}(\lambda_{i}) \subset W_{2}(\lambda_{i}) \subset W_{3}(\lambda_{i}) \subset \cdots $

If you think of the dimension, this can not continue forever. Thus,

$\displaystyle W_{1}(\lambda_{i}) \subset W_{2}(\lambda_{i}) \subset \cdots \subset W_{l}(\lambda_{i}) = W_{l+1}(\lambda_{i}) = \cdots $

and $W(\lambda_{i}) = W_{l}(\lambda_{i})$.

Theorem 5..1  

Let $A$ be the $n$th order square matrix and its characteristic polynomial be (5.1). Then for the generalized eigen space $W(\lambda_{i}) (1 \leq i \leq r)$, the followings hold.

(1) $W(\lambda_{i}) \bigcap W(\lambda_{j}) = {{\bf0}}  (i \neq j)$
(2) Generalized eigenvectors for different eigenvalues are linearly independent
(3) $\dim W(\lambda_{i}) = n_{i}$

Proof (1) For $i \neq j$, there exists a vector ${\mathbf x}$ so that $W(\lambda_{i}) \bigcap W(\lambda_{j}) \ni {\mathbf x} \neq {\bf0}$. Then there is a positive integer $k_{1}$ which satisfies

$\displaystyle (A - \lambda_{i}I)^{k_{1}} {\mathbf x} \neq {\bf0}  \mbox{and}  (A - \lambda_{i})^{k_{1}+1} {\mathbf x} = {\bf0} $

Let ${\mathbf y} = (A - \lambda_{i}I)^{k_{1}}{\mathbf x}$. Then

$\displaystyle (A - \lambda_{i}I) {\mathbf y} = {\bf0},   W(\lambda_{i}) \bigcap W(\lambda_{j}) \ni {\mathbf y} \neq {\bf0}$

Since ${\mathbf y} \in W(\lambda_{j})$, there is a positive integer $k_{2}$ which satisfies $(A - \lambda_{j}I)^{k_{2}} {\mathbf y} = {\bf0}$. But $A{\mathbf y} = \lambda_{i}{\mathbf y}$ implies $(A - \lambda_{j}I)^{k_{2}}{\mathbf y} = (\lambda_{j}I - \lambda_{i}I)^{k_{2}}{\mathbf y} = {\bf0}$. This contradicts the condition that for $i \neq j$, $\lambda_{i} \neq \lambda_{j}$.

(2) Let $\lambda_{1},\lambda_{2},\ldots,\lambda_{s}$ be the distinct eigenvalues of $A$. Then for $W(\lambda_{i}) \ni {\mathbf x}_{i} \neq {\bf0}$ $(1 \leq i \leq s)$, we show by induction that

$\displaystyle c_{1}{\mathbf x}_{1} + c_{2}{\mathbf x}_{2} + \cdots + c_{s}{\mathbf x}_{s} = {\bf0}  \mbox{implies}  c_{1} = c_{2} = \cdots = c_{s} = 0$

For $s = 1$, it is obvious. We assume that

$\displaystyle c_{1}{\mathbf x}_{1} + c_{2}{\mathbf x}_{2} + \cdots + c_{s-1}{\mathbf x}_{s-1} = {\bf0}  \mbox{implies }  c_{1} = c_{2} = \cdots = c_{s-1} = 0$

Then by the condition,

$\displaystyle c_{1}{\mathbf x}_{1} + c_{2}{\mathbf x}_{2} + \cdots + c_{s}{\mathbf x}_{s} = {\bf0}    ** $

We multiply the integer $k$, which satisfies $(A - \lambda_{s}I)^{k} {\mathbf x}_{s} = {\bf0}$, to the equation (**) from the both sides.
$\displaystyle c_{1}(A - \lambda_{s}I)^{k}$   $\displaystyle {\mathbf x}_{1} + \cdots + c_{s-1}(A - \lambda_{s}I)^{k}{\mathbf x}_{s-1} + c_{s}(A - \lambda_{s}I)^{k}{\mathbf x}_{s}$  
  $\displaystyle =$ $\displaystyle c_{1}(A - \lambda_{s}I)^{k}{\mathbf x}_{1} + \cdots + c_{s-1}(A - \lambda_{s}I)^{k}{\mathbf x}_{s-1} = {\bf0}$  

Since $W(\lambda_{i})$ is $(A - \lambda_{s}I)$-invariant, $(A - \lambda_{s}I)^{k}{\mathbf x}_{i} \in W(\lambda_{i})$. Also, from (1), for $i \neq s$, ${\mathbf x}_{i} \not\in W(\lambda_{s})$. Thus, $(A - \lambda_{s}I)^{k}{\mathbf x}_{i} \neq {\bf0}$ $(1 \leq i \leq s-1)$. Then by the induction hypothesis, $c_{i} = 0  (1 \leq i \leq s-1)$. Therefore, $c_{s}{\mathbf x}_{s} = {\bf0}$ and we have $c_{s} = 0$.

(3) by the linearly independence shown (2), we have

$\displaystyle \dim W(\lambda_{1}) + \cdots + \dim W(\lambda_{r}) \leq n$

Also, $n = n_{1} + n_{2} + \cdots + n_{r}$. So, to show (3), it is enough to show $n_{i} \leq \dim W(\lambda_{i})$ $(1 \leq i \leq r)$‚¢.

The matrix $A$ can be transformed to the following triangular matrix by using unitary matrix $U$.

$\displaystyle U^{-1}AU = \left(\begin{array}{rrrrrrrrrr}
\lambda_{1}&&&&&&&&&\\...
...&&&\lambda_{r}&&\\
&&&&&&&&\ddots&\\
&&&&&&&&&\lambda_{r}
\end{array}\right)
$

Now, we let

$\displaystyle V_{i} = \left\{{\mathbf x} = \left(\begin{array}{c}
x_{1}\\
x_...
...ray}\right) \in {\mathcal C}^{n} : x_{i} = 0  (n_{1}+1 \leq i \leq n) \right\}$

Then $V_{1}$ is $n_{1}$th vector subspace of ${\mathcal C}^{n}$. Also, $B = U^{-1}AU$ implies for every vector ${\mathbf x}$ of $V_{1}$, we have

$\displaystyle (B - \lambda_{1}I)^{n_{1}}{\mathbf x} = U^{-1}(A - \lambda_{1}I)^{n_{1}}U{\mathbf x} = {\bf0} $

Note that $U$ is regular,

$\displaystyle (A - \lambda_{1}I)^{n_{1}} U{\mathbf x} - {\bf0} $

Therefore, let $V_{2} = \{U{\mathbf x} : x \in V_{1}\}$. Then $V_{2}$ is $n_{1}$ subspace included in $W(\lambda_{1})$ and obtain the inequality $n_{1} \leq \dim W(\lambda_{1})$. Now switching the order of eigenvalues, we have $n_{i} \leq \dim W(\lambda_{i})$ $(1 \leq i \leq r)$. $ \blacksquare$

Theorem 5..2  

Let $A$ be the $n$th square matrix and its characteristic polynomial be (*). Then $A$ an be transformed by $P$ to the followings.

$\displaystyle P^{-1}AP = \left(\begin{array}{cccc}
A_{1} &&&\\
&A_{2}&&\\
&&&\ddots\\
&&&A_{r}
\end{array}\right)$

Note that $A_{i}(1 \leq i \leq r)$ is $n_{i}$th square matrix and its characteristic polynomial is $\Phi_{A}(t) = (t - \lambda_{i})^{n_{i}}$.

Proof From $W(\lambda_{i})$, we take the basis ${\bf p}_{1}^{i}, \ldots,{\bf p}_{n_{i}}^{i}$. Since $W(\lambda_{i})$ is $A$-invariant, we have

$\displaystyle A{\bf p}_{j}^{i} = a_{1j}^{i}{\vert bf p}_{1}^{i} + \cdots + a_{n_{i}j}^{i}{\bf p}_{n_{i}}^{i}   (1 \leq j \leq n_{i}) $

Now let $A_{i} = (a_{jk}^{i})$. Then $A_{i}$ is $n_{i}$th square matrix. By the theorem 5.1,

$\displaystyle \{{\bf p}_{1}^{1},\ldots,{\bf p}_{n_{1}}^{1},{\bf p}_{1}^{2},\ldots,{\bf p}_{n_{2}}^{2},\ldots,{\bf p}_{1}^{r},\ldots,{\bf p}_{n_{r}}^{r}\} $

is the basis of ${\mathcal C}^{n}$. Therefore, we let

$\displaystyle P = ({\bf p}_{1}^{1} \cdots {\bf p}_{n_{1}}^{1} {\bf p}_{1}^{2} \cdots {\bf p}_{n_{2}}^{2} \cdots {\bf p}_{1}^{r} \cdots {\bf p}_{n_{r}}^{r}) $

Then $P$ is a regular matrix and

$\displaystyle AP = P\left(\begin{array}{cccc}
A_{1} &&&\\
&A_{2}&0&\\
&&&\ddots\\
&&0&A_{r}
\end{array}\right)$

Multiply both sides by $P^{-1}$, we obtain the required equation.

Using this relation,

$\displaystyle \Phi_{A}(t)$ $\displaystyle =$ $\displaystyle \Phi_{P^{-1}AP}(t) = (t - \lambda_{1})^{n_{1}}(t - \lambda_{2})^{n_{2}}\cdots(t - \lambda_{r})^{n_{r}}$  
  $\displaystyle =$ $\displaystyle \Phi_{A_{1}}(t)\Phi_{A_{2}}(t) \cdots \Phi_{A_{r}}(t)$  

Therefore, if we can show that the eigenvalue of $A_{i}$ is $\lambda_{i}$ only. Then we can show

$\displaystyle \Phi_{A_{i}}(t) = (t - \lambda_{i})^{n_{i}}$

and $A_{i}$ has the eigenvalues $\lambda_{i}(j \neq i)$ and the eigenvector corresponds to $\lambda_{i}$ are

$\displaystyle {\mathbf x} = \left(\begin{array}{c}
x_{1}\\
x_{2}\\
\vdots\...
...}\right) = \left(\begin{array}{c}
0\\
0\\
\vdots\\
0
\end{array}\right)$

In other words, if $A_{i}{\mathbf x} = \lambda_{j}{\mathbf x}$ and let

$\displaystyle {\mathbf {\bar{x}}} = \left(\begin{array}{c}
0\\
\vdots\\
0\...
...
\\
\\
\end{array}\right\} & n_{i+1} + \cdots + n_{r}
\end{array}\right.$

Then ${\mathbf {\bar x}} \neq {\bf0}$‚Å $P^{-1}AP{\mathbf {\bar x}} = \lambda_{j}{\mathbf {\bar x}}$. Therefore,

$\displaystyle {\bf0} \neq P{\mathbf {\bar x}} = x_{1}{\bf p}_{1}^{i} + \cdots + x_{n_{i}}{\bf p}_{n_{i}}^{i} \in W(\lambda_{i}) $

satisfies $AP{\mathbf {\bar x}} = \lambda_{j}P{\mathbf {\bar x}}$. In other words, it satisfies $(A - \lambda_{j}I)P{\mathbf {\bar x}} = {\bf0}$. But then this means that $P{\mathbf {\bar x}} \in W(\lambda_{j})$ and this contradicts the theorem 5.1. $ \blacksquare$